How to prove the inequality $a/(b+c)+b/(a+c)+c/(a+b) ge 3/2$ [duplicate] The 2019 Stack Overflow Developer Survey Results Are InProof of the inequality $fracab+c+fracba+c+fracca+b geq frac32$Proof of Nesbitt's Inequality: $fracab+c+fracbc+a+fracca+bge frac32$?how to prove this inequality?how to solve a inequality?Prove this inequality with $xyzle 1$How to show the inequality is strict?An inequality about the sum of distances between points : same color $le$ different colors?How to prove this inequalityMaybe this inequality holds? $x!-y!>x^n$?Prove this by inequality with four variables inequalityA tricky integral inequalityClueless as to how to solve this gamma function differential inequality

How to change the limits of integration

I looked up a future colleague on LinkedIn before I started a job. I told my colleague about it and he seemed surprised. Should I apologize?

What is the meaning of Triage in Cybersec world?

Is there a name of the flying bionic bird?

Is bread bad for ducks?

Understanding the implication of what "well-defined" means for the operation in quotient group

What is the purpose of the constant in the probability density function

Inline version of a function returns different value than non-inline version

Does a dangling wire really electrocute me if I'm standing in water?

Why is my p-value correlated to difference between means in two sample tests?

What is the motivation for a law requiring 2 parties to consent for recording a conversation

How to deal with fear of taking dependencies

How to create dashed lines/arrows in Illustrator

A poker game description that does not feel gimmicky

How to manage monthly salary

JSON.serialize: is it possible to suppress null values of a map?

How to answer pointed "are you quitting" questioning when I don't want them to suspect

Deadlock Graph and Interpretation, solution to avoid

The difference between dialogue marks

Pristine Bit Checking

Access elements in std::string where positon of string is greater than its size

Is flight data recorder erased after every flight?

Why is Grand Jury testimony secret?

How was Skylab's orbit inclination chosen?



How to prove the inequality $a/(b+c)+b/(a+c)+c/(a+b) ge 3/2$ [duplicate]



The 2019 Stack Overflow Developer Survey Results Are InProof of the inequality $fracab+c+fracba+c+fracca+b geq frac32$Proof of Nesbitt's Inequality: $fracab+c+fracbc+a+fracca+bge frac32$?how to prove this inequality?how to solve a inequality?Prove this inequality with $xyzle 1$How to show the inequality is strict?An inequality about the sum of distances between points : same color $le$ different colors?How to prove this inequalityMaybe this inequality holds? $x!-y!>x^n$?Prove this by inequality with four variables inequalityA tricky integral inequalityClueless as to how to solve this gamma function differential inequality










1












$begingroup$



This question already has an answer here:



  • Proof of the inequality $fracab+c+fracba+c+fracca+b geq frac32$

    5 answers



Suppose $a>0, b>0, c>0$.



Prove that:
$$a+b+c ge frac32cdot [(a+b)(a+c)(b+c)]^frac13$$




Hence or otherwise prove:
$$colorbluefracab+c+fracba+c+fracca+bge frac32$$











share|cite|improve this question











$endgroup$



marked as duplicate by Lord Shark the Unknown, Javi, darij grinberg, Lee David Chung Lin, Tianlalu Apr 3 at 1:35


This question has been asked before and already has an answer. If those answers do not fully address your question, please ask a new question.

















  • $begingroup$
    The final inequality you want to prove is known as Nesbitt's inequality. See the answers here or here for proofs.
    $endgroup$
    – Minus One-Twelfth
    Mar 30 at 11:50











  • $begingroup$
    Also, the first inequality you are asked to prove follows from the AM-GM inequality ($fracx+y+z3ge sqrt[3]xyz$ for $x,y,zge 0$).
    $endgroup$
    – Minus One-Twelfth
    Mar 30 at 11:56
















1












$begingroup$



This question already has an answer here:



  • Proof of the inequality $fracab+c+fracba+c+fracca+b geq frac32$

    5 answers



Suppose $a>0, b>0, c>0$.



Prove that:
$$a+b+c ge frac32cdot [(a+b)(a+c)(b+c)]^frac13$$




Hence or otherwise prove:
$$colorbluefracab+c+fracba+c+fracca+bge frac32$$











share|cite|improve this question











$endgroup$



marked as duplicate by Lord Shark the Unknown, Javi, darij grinberg, Lee David Chung Lin, Tianlalu Apr 3 at 1:35


This question has been asked before and already has an answer. If those answers do not fully address your question, please ask a new question.

















  • $begingroup$
    The final inequality you want to prove is known as Nesbitt's inequality. See the answers here or here for proofs.
    $endgroup$
    – Minus One-Twelfth
    Mar 30 at 11:50











  • $begingroup$
    Also, the first inequality you are asked to prove follows from the AM-GM inequality ($fracx+y+z3ge sqrt[3]xyz$ for $x,y,zge 0$).
    $endgroup$
    – Minus One-Twelfth
    Mar 30 at 11:56














1












1








1





$begingroup$



This question already has an answer here:



  • Proof of the inequality $fracab+c+fracba+c+fracca+b geq frac32$

    5 answers



Suppose $a>0, b>0, c>0$.



Prove that:
$$a+b+c ge frac32cdot [(a+b)(a+c)(b+c)]^frac13$$




Hence or otherwise prove:
$$colorbluefracab+c+fracba+c+fracca+bge frac32$$











share|cite|improve this question











$endgroup$





This question already has an answer here:



  • Proof of the inequality $fracab+c+fracba+c+fracca+b geq frac32$

    5 answers



Suppose $a>0, b>0, c>0$.



Prove that:
$$a+b+c ge frac32cdot [(a+b)(a+c)(b+c)]^frac13$$




Hence or otherwise prove:
$$colorbluefracab+c+fracba+c+fracca+bge frac32$$






This question already has an answer here:



  • Proof of the inequality $fracab+c+fracba+c+fracca+b geq frac32$

    5 answers







inequality






share|cite|improve this question















share|cite|improve this question













share|cite|improve this question




share|cite|improve this question








edited Mar 30 at 16:54









Dr. Mathva

3,493630




3,493630










asked Mar 30 at 11:47









M.YouM.You

21




21




marked as duplicate by Lord Shark the Unknown, Javi, darij grinberg, Lee David Chung Lin, Tianlalu Apr 3 at 1:35


This question has been asked before and already has an answer. If those answers do not fully address your question, please ask a new question.









marked as duplicate by Lord Shark the Unknown, Javi, darij grinberg, Lee David Chung Lin, Tianlalu Apr 3 at 1:35


This question has been asked before and already has an answer. If those answers do not fully address your question, please ask a new question.













  • $begingroup$
    The final inequality you want to prove is known as Nesbitt's inequality. See the answers here or here for proofs.
    $endgroup$
    – Minus One-Twelfth
    Mar 30 at 11:50











  • $begingroup$
    Also, the first inequality you are asked to prove follows from the AM-GM inequality ($fracx+y+z3ge sqrt[3]xyz$ for $x,y,zge 0$).
    $endgroup$
    – Minus One-Twelfth
    Mar 30 at 11:56

















  • $begingroup$
    The final inequality you want to prove is known as Nesbitt's inequality. See the answers here or here for proofs.
    $endgroup$
    – Minus One-Twelfth
    Mar 30 at 11:50











  • $begingroup$
    Also, the first inequality you are asked to prove follows from the AM-GM inequality ($fracx+y+z3ge sqrt[3]xyz$ for $x,y,zge 0$).
    $endgroup$
    – Minus One-Twelfth
    Mar 30 at 11:56
















$begingroup$
The final inequality you want to prove is known as Nesbitt's inequality. See the answers here or here for proofs.
$endgroup$
– Minus One-Twelfth
Mar 30 at 11:50





$begingroup$
The final inequality you want to prove is known as Nesbitt's inequality. See the answers here or here for proofs.
$endgroup$
– Minus One-Twelfth
Mar 30 at 11:50













$begingroup$
Also, the first inequality you are asked to prove follows from the AM-GM inequality ($fracx+y+z3ge sqrt[3]xyz$ for $x,y,zge 0$).
$endgroup$
– Minus One-Twelfth
Mar 30 at 11:56





$begingroup$
Also, the first inequality you are asked to prove follows from the AM-GM inequality ($fracx+y+z3ge sqrt[3]xyz$ for $x,y,zge 0$).
$endgroup$
– Minus One-Twelfth
Mar 30 at 11:56











4 Answers
4






active

oldest

votes


















2












$begingroup$

Using AM-GM inequality:$$fracx_1+...+x_nngeq (x_1...x_n)^1/n$$
let $n=3$ and $x_1=a+b,x_2=b+c,x_3=c+a$:
$$frac2(a+b+c)3geq [(a+b)(b+c)(c+a)]^1/3$$






share|cite|improve this answer









$endgroup$




















    1












    $begingroup$

    Hint: Substitute $$b+c=x,a+c=y,a+b=z$$ so $$a=frac-x+y+z2$$
    $$b=fracx-y+z2$$
    $$c=fracx+y-z2$$
    And we get
    $$frac-x+y+z2x+fracx-y+z2y+fracx+y-z2zgeq frac32$$
    Can you finish?
    And we get $$fracxy+fracyx+fracyz+fraczy+fracxz+fraczxgeq 6$$






    share|cite|improve this answer











    $endgroup$




















      1












      $begingroup$

      To minimize $fracab+c+fracbc+a+fracca+b$ , we need to have
      $$
      beginalign
      0
      &=deltaleft(fracab+c+fracbc+a+fracca+bright)\
      &=left(frac1b+c-fracb(c+a)^2-fracc(a+b)^2right)delta a\
      &+left(frac1c+a-fracc(a+b)^2-fraca(b+c)^2right)delta b\
      &+left(frac1a+b-fraca(b+c)^2-fracb(c+a)^2right)delta ctag1
      endalign
      $$

      for all $delta a,delta b,delta c$. That means
      $$
      beginalign
      frac1a+b&=fraca(b+c)^2+fracb(c+a)^2tag2\
      frac1b+c&=fracb(c+a)^2+fracc(a+b)^2tag3\
      frac1c+a&=fracc(a+b)^2+fraca(b+c)^2tag4
      endalign
      $$

      Subtract $(4)$ from the sum of $(2)$ and $(3)$:
      $$
      frac1b+c+frac1a+b-frac1c+a=frac2b(c+a)^2tag5
      $$

      Add $frac2c+a$ and divide by $2(a+b+c)$:
      $$
      frac12(a+b+c)left(frac1b+c+frac1a+b+frac1c+aright)=frac1(c+a)^2tag6
      $$

      By symmetry,
      $$
      frac1(a+b)^2=frac1(b+c)^2=frac1(c+a)^2tag7
      $$

      from which we get $a=b=c$. Thus, we get
      $$
      fracab+c+fracbc+a+fracca+bgefrac32tag8
      $$






      share|cite|improve this answer









      $endgroup$




















        0












        $begingroup$

        Hint:



        $$fracab+c+fracba+c+fracca+bge frac32iff fraca+b+cb+c+fracb+a+ca+c+fracc+a+ba+bge frac32+3$$



        $$iff big(a+b+cbig)cdot bigg(frac1b+c+frac1a+c+frac1a+bbigg)ge frac92iff colorbluefrac2cdot (a+b+c)3ge frac3frac1b+c+frac1a+c+frac1a+b$$ Which is trivial by the AM-HM inequality. Done!






        share|cite|improve this answer









        $endgroup$



















          4 Answers
          4






          active

          oldest

          votes








          4 Answers
          4






          active

          oldest

          votes









          active

          oldest

          votes






          active

          oldest

          votes









          2












          $begingroup$

          Using AM-GM inequality:$$fracx_1+...+x_nngeq (x_1...x_n)^1/n$$
          let $n=3$ and $x_1=a+b,x_2=b+c,x_3=c+a$:
          $$frac2(a+b+c)3geq [(a+b)(b+c)(c+a)]^1/3$$






          share|cite|improve this answer









          $endgroup$

















            2












            $begingroup$

            Using AM-GM inequality:$$fracx_1+...+x_nngeq (x_1...x_n)^1/n$$
            let $n=3$ and $x_1=a+b,x_2=b+c,x_3=c+a$:
            $$frac2(a+b+c)3geq [(a+b)(b+c)(c+a)]^1/3$$






            share|cite|improve this answer









            $endgroup$















              2












              2








              2





              $begingroup$

              Using AM-GM inequality:$$fracx_1+...+x_nngeq (x_1...x_n)^1/n$$
              let $n=3$ and $x_1=a+b,x_2=b+c,x_3=c+a$:
              $$frac2(a+b+c)3geq [(a+b)(b+c)(c+a)]^1/3$$






              share|cite|improve this answer









              $endgroup$



              Using AM-GM inequality:$$fracx_1+...+x_nngeq (x_1...x_n)^1/n$$
              let $n=3$ and $x_1=a+b,x_2=b+c,x_3=c+a$:
              $$frac2(a+b+c)3geq [(a+b)(b+c)(c+a)]^1/3$$







              share|cite|improve this answer












              share|cite|improve this answer



              share|cite|improve this answer










              answered Mar 30 at 11:59









              Xin FuXin Fu

              34319




              34319





















                  1












                  $begingroup$

                  Hint: Substitute $$b+c=x,a+c=y,a+b=z$$ so $$a=frac-x+y+z2$$
                  $$b=fracx-y+z2$$
                  $$c=fracx+y-z2$$
                  And we get
                  $$frac-x+y+z2x+fracx-y+z2y+fracx+y-z2zgeq frac32$$
                  Can you finish?
                  And we get $$fracxy+fracyx+fracyz+fraczy+fracxz+fraczxgeq 6$$






                  share|cite|improve this answer











                  $endgroup$

















                    1












                    $begingroup$

                    Hint: Substitute $$b+c=x,a+c=y,a+b=z$$ so $$a=frac-x+y+z2$$
                    $$b=fracx-y+z2$$
                    $$c=fracx+y-z2$$
                    And we get
                    $$frac-x+y+z2x+fracx-y+z2y+fracx+y-z2zgeq frac32$$
                    Can you finish?
                    And we get $$fracxy+fracyx+fracyz+fraczy+fracxz+fraczxgeq 6$$






                    share|cite|improve this answer











                    $endgroup$















                      1












                      1








                      1





                      $begingroup$

                      Hint: Substitute $$b+c=x,a+c=y,a+b=z$$ so $$a=frac-x+y+z2$$
                      $$b=fracx-y+z2$$
                      $$c=fracx+y-z2$$
                      And we get
                      $$frac-x+y+z2x+fracx-y+z2y+fracx+y-z2zgeq frac32$$
                      Can you finish?
                      And we get $$fracxy+fracyx+fracyz+fraczy+fracxz+fraczxgeq 6$$






                      share|cite|improve this answer











                      $endgroup$



                      Hint: Substitute $$b+c=x,a+c=y,a+b=z$$ so $$a=frac-x+y+z2$$
                      $$b=fracx-y+z2$$
                      $$c=fracx+y-z2$$
                      And we get
                      $$frac-x+y+z2x+fracx-y+z2y+fracx+y-z2zgeq frac32$$
                      Can you finish?
                      And we get $$fracxy+fracyx+fracyz+fraczy+fracxz+fraczxgeq 6$$







                      share|cite|improve this answer














                      share|cite|improve this answer



                      share|cite|improve this answer








                      edited Mar 30 at 12:02

























                      answered Mar 30 at 11:54









                      Dr. Sonnhard GraubnerDr. Sonnhard Graubner

                      78.8k42867




                      78.8k42867





















                          1












                          $begingroup$

                          To minimize $fracab+c+fracbc+a+fracca+b$ , we need to have
                          $$
                          beginalign
                          0
                          &=deltaleft(fracab+c+fracbc+a+fracca+bright)\
                          &=left(frac1b+c-fracb(c+a)^2-fracc(a+b)^2right)delta a\
                          &+left(frac1c+a-fracc(a+b)^2-fraca(b+c)^2right)delta b\
                          &+left(frac1a+b-fraca(b+c)^2-fracb(c+a)^2right)delta ctag1
                          endalign
                          $$

                          for all $delta a,delta b,delta c$. That means
                          $$
                          beginalign
                          frac1a+b&=fraca(b+c)^2+fracb(c+a)^2tag2\
                          frac1b+c&=fracb(c+a)^2+fracc(a+b)^2tag3\
                          frac1c+a&=fracc(a+b)^2+fraca(b+c)^2tag4
                          endalign
                          $$

                          Subtract $(4)$ from the sum of $(2)$ and $(3)$:
                          $$
                          frac1b+c+frac1a+b-frac1c+a=frac2b(c+a)^2tag5
                          $$

                          Add $frac2c+a$ and divide by $2(a+b+c)$:
                          $$
                          frac12(a+b+c)left(frac1b+c+frac1a+b+frac1c+aright)=frac1(c+a)^2tag6
                          $$

                          By symmetry,
                          $$
                          frac1(a+b)^2=frac1(b+c)^2=frac1(c+a)^2tag7
                          $$

                          from which we get $a=b=c$. Thus, we get
                          $$
                          fracab+c+fracbc+a+fracca+bgefrac32tag8
                          $$






                          share|cite|improve this answer









                          $endgroup$

















                            1












                            $begingroup$

                            To minimize $fracab+c+fracbc+a+fracca+b$ , we need to have
                            $$
                            beginalign
                            0
                            &=deltaleft(fracab+c+fracbc+a+fracca+bright)\
                            &=left(frac1b+c-fracb(c+a)^2-fracc(a+b)^2right)delta a\
                            &+left(frac1c+a-fracc(a+b)^2-fraca(b+c)^2right)delta b\
                            &+left(frac1a+b-fraca(b+c)^2-fracb(c+a)^2right)delta ctag1
                            endalign
                            $$

                            for all $delta a,delta b,delta c$. That means
                            $$
                            beginalign
                            frac1a+b&=fraca(b+c)^2+fracb(c+a)^2tag2\
                            frac1b+c&=fracb(c+a)^2+fracc(a+b)^2tag3\
                            frac1c+a&=fracc(a+b)^2+fraca(b+c)^2tag4
                            endalign
                            $$

                            Subtract $(4)$ from the sum of $(2)$ and $(3)$:
                            $$
                            frac1b+c+frac1a+b-frac1c+a=frac2b(c+a)^2tag5
                            $$

                            Add $frac2c+a$ and divide by $2(a+b+c)$:
                            $$
                            frac12(a+b+c)left(frac1b+c+frac1a+b+frac1c+aright)=frac1(c+a)^2tag6
                            $$

                            By symmetry,
                            $$
                            frac1(a+b)^2=frac1(b+c)^2=frac1(c+a)^2tag7
                            $$

                            from which we get $a=b=c$. Thus, we get
                            $$
                            fracab+c+fracbc+a+fracca+bgefrac32tag8
                            $$






                            share|cite|improve this answer









                            $endgroup$















                              1












                              1








                              1





                              $begingroup$

                              To minimize $fracab+c+fracbc+a+fracca+b$ , we need to have
                              $$
                              beginalign
                              0
                              &=deltaleft(fracab+c+fracbc+a+fracca+bright)\
                              &=left(frac1b+c-fracb(c+a)^2-fracc(a+b)^2right)delta a\
                              &+left(frac1c+a-fracc(a+b)^2-fraca(b+c)^2right)delta b\
                              &+left(frac1a+b-fraca(b+c)^2-fracb(c+a)^2right)delta ctag1
                              endalign
                              $$

                              for all $delta a,delta b,delta c$. That means
                              $$
                              beginalign
                              frac1a+b&=fraca(b+c)^2+fracb(c+a)^2tag2\
                              frac1b+c&=fracb(c+a)^2+fracc(a+b)^2tag3\
                              frac1c+a&=fracc(a+b)^2+fraca(b+c)^2tag4
                              endalign
                              $$

                              Subtract $(4)$ from the sum of $(2)$ and $(3)$:
                              $$
                              frac1b+c+frac1a+b-frac1c+a=frac2b(c+a)^2tag5
                              $$

                              Add $frac2c+a$ and divide by $2(a+b+c)$:
                              $$
                              frac12(a+b+c)left(frac1b+c+frac1a+b+frac1c+aright)=frac1(c+a)^2tag6
                              $$

                              By symmetry,
                              $$
                              frac1(a+b)^2=frac1(b+c)^2=frac1(c+a)^2tag7
                              $$

                              from which we get $a=b=c$. Thus, we get
                              $$
                              fracab+c+fracbc+a+fracca+bgefrac32tag8
                              $$






                              share|cite|improve this answer









                              $endgroup$



                              To minimize $fracab+c+fracbc+a+fracca+b$ , we need to have
                              $$
                              beginalign
                              0
                              &=deltaleft(fracab+c+fracbc+a+fracca+bright)\
                              &=left(frac1b+c-fracb(c+a)^2-fracc(a+b)^2right)delta a\
                              &+left(frac1c+a-fracc(a+b)^2-fraca(b+c)^2right)delta b\
                              &+left(frac1a+b-fraca(b+c)^2-fracb(c+a)^2right)delta ctag1
                              endalign
                              $$

                              for all $delta a,delta b,delta c$. That means
                              $$
                              beginalign
                              frac1a+b&=fraca(b+c)^2+fracb(c+a)^2tag2\
                              frac1b+c&=fracb(c+a)^2+fracc(a+b)^2tag3\
                              frac1c+a&=fracc(a+b)^2+fraca(b+c)^2tag4
                              endalign
                              $$

                              Subtract $(4)$ from the sum of $(2)$ and $(3)$:
                              $$
                              frac1b+c+frac1a+b-frac1c+a=frac2b(c+a)^2tag5
                              $$

                              Add $frac2c+a$ and divide by $2(a+b+c)$:
                              $$
                              frac12(a+b+c)left(frac1b+c+frac1a+b+frac1c+aright)=frac1(c+a)^2tag6
                              $$

                              By symmetry,
                              $$
                              frac1(a+b)^2=frac1(b+c)^2=frac1(c+a)^2tag7
                              $$

                              from which we get $a=b=c$. Thus, we get
                              $$
                              fracab+c+fracbc+a+fracca+bgefrac32tag8
                              $$







                              share|cite|improve this answer












                              share|cite|improve this answer



                              share|cite|improve this answer










                              answered Mar 30 at 17:45









                              robjohnrobjohn

                              270k27313642




                              270k27313642





















                                  0












                                  $begingroup$

                                  Hint:



                                  $$fracab+c+fracba+c+fracca+bge frac32iff fraca+b+cb+c+fracb+a+ca+c+fracc+a+ba+bge frac32+3$$



                                  $$iff big(a+b+cbig)cdot bigg(frac1b+c+frac1a+c+frac1a+bbigg)ge frac92iff colorbluefrac2cdot (a+b+c)3ge frac3frac1b+c+frac1a+c+frac1a+b$$ Which is trivial by the AM-HM inequality. Done!






                                  share|cite|improve this answer









                                  $endgroup$

















                                    0












                                    $begingroup$

                                    Hint:



                                    $$fracab+c+fracba+c+fracca+bge frac32iff fraca+b+cb+c+fracb+a+ca+c+fracc+a+ba+bge frac32+3$$



                                    $$iff big(a+b+cbig)cdot bigg(frac1b+c+frac1a+c+frac1a+bbigg)ge frac92iff colorbluefrac2cdot (a+b+c)3ge frac3frac1b+c+frac1a+c+frac1a+b$$ Which is trivial by the AM-HM inequality. Done!






                                    share|cite|improve this answer









                                    $endgroup$















                                      0












                                      0








                                      0





                                      $begingroup$

                                      Hint:



                                      $$fracab+c+fracba+c+fracca+bge frac32iff fraca+b+cb+c+fracb+a+ca+c+fracc+a+ba+bge frac32+3$$



                                      $$iff big(a+b+cbig)cdot bigg(frac1b+c+frac1a+c+frac1a+bbigg)ge frac92iff colorbluefrac2cdot (a+b+c)3ge frac3frac1b+c+frac1a+c+frac1a+b$$ Which is trivial by the AM-HM inequality. Done!






                                      share|cite|improve this answer









                                      $endgroup$



                                      Hint:



                                      $$fracab+c+fracba+c+fracca+bge frac32iff fraca+b+cb+c+fracb+a+ca+c+fracc+a+ba+bge frac32+3$$



                                      $$iff big(a+b+cbig)cdot bigg(frac1b+c+frac1a+c+frac1a+bbigg)ge frac92iff colorbluefrac2cdot (a+b+c)3ge frac3frac1b+c+frac1a+c+frac1a+b$$ Which is trivial by the AM-HM inequality. Done!







                                      share|cite|improve this answer












                                      share|cite|improve this answer



                                      share|cite|improve this answer










                                      answered Mar 30 at 17:13









                                      Dr. MathvaDr. Mathva

                                      3,493630




                                      3,493630













                                          Popular posts from this blog

                                          Triangular numbers and gcdProving sum of a set is $0 pmod n$ if $n$ is odd, or $fracn2 pmod n$ if $n$ is even?Is greatest common divisor of two numbers really their smallest linear combination?GCD, LCM RelationshipProve a set of nonnegative integers with greatest common divisor 1 and closed under addition has all but finite many nonnegative integers.all pairs of a and b in an equation containing gcdTriangular Numbers Modulo $k$ - Hit All Values?Understanding the Existence and Uniqueness of the GCDGCD and LCM with logical symbolsThe greatest common divisor of two positive integers less than 100 is equal to 3. Their least common multiple is twelve times one of the integers.Suppose that for all integers $x$, $x|a$ and $x|b$ if and only if $x|c$. Then $c = gcd(a,b)$Which is the gcd of 2 numbers which are multiplied and the result is 600000?

                                          Ingelân Ynhâld Etymology | Geografy | Skiednis | Polityk en bestjoer | Ekonomy | Demografy | Kultuer | Klimaat | Sjoch ek | Keppelings om utens | Boarnen, noaten en referinsjes Navigaasjemenuwww.gov.ukOffisjele webside fan it regear fan it Feriene KeninkrykOffisjele webside fan it Britske FerkearsburoNederlânsktalige ynformaasje fan it Britske FerkearsburoOffisjele webside fan English Heritage, de organisaasje dy't him ynset foar it behâld fan it Ingelske kultuergoedYnwennertallen fan alle Britske stêden út 'e folkstelling fan 2011Notes en References, op dizze sideEngland

                                          Հադիս Բովանդակություն Անվանում և նշանակություն | Դասակարգում | Աղբյուրներ | Նավարկման ցանկ